Music historian: Some critics lament the fact that impoverished postwar recording studios forced early bebop musicia...

on November 1, 2017

Help

Can you explain the correct answer

Replies
Create a free account to read and take part in forum discussions.

Already have an account? log in

Jameka on January 15, 2019

It would be great to get an explanation here. I selected answer choice E.

Ravi on January 15, 2019

@franco and @JCarter,

Happy to help! The question is asking us to pick the answer that the
music historian's statements most strongly support. Another way of
thinking about this question is that we're choosing the answer that
would best serve as the conclusion for what's in the stimulus.

Answer A is incorrect because we have no information in the stimulus
that tells us about the value of artistic works. We can get rid of
this.

Answer B says, "the difficult postwar recording conditions had some
beneficial consequences for bebop." This can be supported by the
stimulus, as the historian says, "these musicians' beautifully concise
playing makes the recordings superb artistic works," also noting that
the conciseness of these recordings fostered a compactness that the
next generation lacks. This implies that there were some positive
things to come out of the recording constraints, and this supports
answer B well.

Answer C is incorrect because the stimulus makes no mention of
comparing all bebop recordings. This statement is far too strong.

Answer D is incorrect because the stimulus only discusses one facet of
music in comparing bebop to the next generation; we don't have nearly
enough support from the stimulus to say that the next generation's
music was of lower quality. This answer choice is out.

Answer E is incorrect we only have one instance in which conditions
forced musicians to record short solos. E is saying that if a musician
records a short solo - ->forced to do so by difficult recording
conditions. Who knows, musicians could voluntarily choose to record
extremely short solos sometimes. This statement is far too strong and
can't be supported by the stimulus.

Hope this helps. Let us know if you have any more questions!